Đến nội dung

quanganhthanhhoa nội dung

Có 35 mục bởi quanganhthanhhoa (Tìm giới hạn từ 29-04-2020)



Sắp theo                Sắp xếp  

#609545 Tiếp sức bất đẳng thức

Đã gửi bởi quanganhthanhhoa on 17-01-2016 - 21:41 trong Bất đẳng thức và cực trị

Ta có: 

$P=x(1-x^{2})\Rightarrow P^{2}=x^{2}(1-x^{2})^{2}$

$P^{2}=\frac{1}{2}2x^{2}(1-x^{2})(1-x^{2})\leq \frac{1}{2}\frac{(2)^{3}}{27}=\frac{4}{27}$

$\Rightarrow P\leq \frac{2}{3\sqrt{3}}$

Dấu "=" xảy ra $\Leftrightarrow x=\sqrt{\frac{1}{3}}; y=\sqrt{\frac{2}{3}}$

Vậy Pmax=$\frac{2}{3\sqrt{3}}$

Mình bổ sung thêm bài toán mới

Bài 10:Cho $a,b,c>0$ và $a+b+c=3$.Chứng minh $\sum \frac{a^{3}}{(2a^2+b^2)(2a^2+c^2)}\geq \frac{1}{3}$




#609319 Tiếp sức bất đẳng thức

Đã gửi bởi quanganhthanhhoa on 16-01-2016 - 21:20 trong Bất đẳng thức và cực trị

Bài 1:Biến đổi tương đương ta có $\frac{1}{2}(x-y)(x-z)(y-z)[(x+y)^{2}+(y+z)^{2}+(z+x)^{2}]\geq 0$(luôn đúng)

Dấu ''='' xảy ra khi $x=y=z$

Xin đề xuất bài toán tiếp theo:

Bài 3:Cho $a,b,c>0$.Chứng minh $\sum \frac{a^{3}+b^{3}}{2ab}\geq a+b+c$

Nhân tiện cho chúc mừng sinh nhật VMF ké nhé  :D




#612266 Tiếp sức bất đẳng thức

Đã gửi bởi quanganhthanhhoa on 01-02-2016 - 18:06 trong Bất đẳng thức và cực trị

Gợi ý:Chứng minh bất đẳng thức sau $\frac{1}{4+a-ab}+\frac{1}{4+b-bc}+\frac{1}{4+c-ac}\geq \frac{3}{3+abc}$

Không cần phức tạp thế đâu bạn tôi ơi  :D

$\sum \frac{1}{4+a-ab}=\sum \frac{1}{4+a(1-b)}$

Do $(a,b,c)\epsilon [1;2]\Rightarrow \frac{3}{2}\leq \sum \frac{1}{4+a(1-b)}\leq \frac{3}{4}$

Dấu ''='' của Max xảy ra tại $a=b=c=2$

Dấu ''='' của Min là $a=b=c=1$ 

Có bài này cũng hay mà đơn giản nè:

Bài 14:Cho $a,b,c$ không âm thỏa mãn $ab+bc+ca=3$.Chứng  minh rằng $a^3+b^3+c^3+6abc \geq 9$

P/s:Góp ý chút,bài nào giải rồi nên tô đỏ để mọi người biết là đã được giải :)




#613087 Tiếp sức bất đẳng thức

Đã gửi bởi quanganhthanhhoa on 05-02-2016 - 18:33 trong Bất đẳng thức và cực trị

Muốn hỏi mấy bài này

Bài 19: Cho $a,b,c \geq 1$.Tìm Max:$A=a+b+c+ab+bc+ca-3abc$

Bài 20: Cho $a,b,c \geq 1$.Tìm Max $\sum \frac{1}{ab+a+1}$




#613431 Tiếp sức bất đẳng thức

Đã gửi bởi quanganhthanhhoa on 07-02-2016 - 10:07 trong Bất đẳng thức và cực trị

Bài 36. Cho a,b,c là các số thực dương thỏa mãn a.b.c = 1

Tìm Max của $\frac{a}{b^{2} + c^{2} + a} + \frac{b}{a^{2} + c^{2} + b} + \frac{c}{a^{2} + b^{2} + c}$

$\sum \frac{a}{b^{2} + c^{2} + a} =\sum \frac{a^3}{b^{6} + c^{6} + a^3} \leq \sum \frac{a^{3}}{bc(b^4+c^4)+a^4bc}=\sum \frac{a^{3}}{\frac{b^4+c^4}{a}+a^3}=\sum \frac{a^{4}}{a^4+b^4+c^4}=1$

DBXR khi $a=b=c=1$

Bài 40: Cho $x,y>0$ thỏa mãn $\sqrt{\frac{2x}{y}}(2xy-1)=2xy+1$.Tìm Min:$2x+\frac{1}{y}$

Bài 41: Cho $a,b,c,d>0$ thỏa mãn $(a+b)(c+d)\geq 4abcd$.Chứng minh $\frac{1}{ab(c+1)}+\frac{1}{bc(d+1)}+\frac{1}{cd(a+1)}+\frac{1}{ad(b+1)}\geq \frac{32}{(a+1)(b+1)(c+1)(d+1)}$




#606963 Tổng hợp các bài BĐT

Đã gửi bởi quanganhthanhhoa on 03-01-2016 - 15:28 trong Bất đẳng thức và cực trị

Giúp mình  bài này:

Cho $a, b, c$ là các số thực dương thỏa mãn $a+b+c=1$ . Chứng minh rằng :
$\dfrac{b\sqrt{c}}{a\left (\sqrt{3c}+\sqrt{ab}\right )}+\dfrac{c\sqrt{a}}{b\left (\sqrt{3a}+\sqrt{bc}\right )}+\dfrac{a\sqrt{b}}{c\left (\sqrt{3b}+\sqrt{ca}\right )}\ge \dfrac{3\sqrt{3}}{4}$




#608224 Topic Ôn thi HSG 9 2015-2016 (Hình học)

Đã gửi bởi quanganhthanhhoa on 09-01-2016 - 21:52 trong Chuyên đề toán THCS

Bài 68:Cho đường tròn (O,R) và dây $AB$ cố định ($AB$ không là đường kính).Từ điểm P di động trên tia đối của tia AB vẽ 2 tiếp tuyến $PN,PQ$.Gọi $K$ là giao của $OP$ và $NQ$.Chứng minh $K$ luôn nằm trên 1 đường tròn cố định




#617080 Topic Ôn thi HSG 9 2015-2016 (Hình học)

Đã gửi bởi quanganhthanhhoa on 26-02-2016 - 20:22 trong Chuyên đề toán THCS

 

 

Bài $25$: Cho $\triangle \ ABC$, các phân giác $AD, BE. CF$, $M$ là giao điểm $BE$ và $DF$, $N$ là giao điểm $DE$ và $CF$. Chứng minh $\widehat{FAM}=\widehat{EAN}$

Nhờ bạn xem hộ lại bài 25,mình vẽ hình trên GeoGeobra không ra được 2 góc cần cm bằng nhau !




#606961 TOPIC:CÁC CHUYÊN ĐỀ SỐ HỌC ÔN THI HSG TOÁN 9 VÀ VÀO LỚP 10

Đã gửi bởi quanganhthanhhoa on 03-01-2016 - 15:20 trong Số học

Giúp mình bài này:

Bài 22:Tồn tại hay không số tự nhiên $x$ thỏa mãn:

a)$x^2+x+1 \vdots 31$

b)$x^2+x+1 \vdots 2017$

c)$x^2+x+1 \vdots 5$




#617448 Đề kiểm tra đội tuyển toán THPT chuyên Hà Nội Amsterdam -Vòng 2-Năm học 2015-...

Đã gửi bởi quanganhthanhhoa on 28-02-2016 - 17:29 trong Tài liệu - Đề thi

 

$\frac{1}{a}+\frac{1}{b}=\frac{1}{c}\Leftrightarrow (a+b)c=ab$

Gỉa sử $a+b$ là số nguyên tố. Suy ra $a+b=ab$(vi $a+b>c)

$\Leftrightarrow (a-1)(b-1)=1$$\Leftrightarrow a=b=2$

  $a+b=4$. Vậy a+b ko thể là SNT

Đoạn này sai,từ $a+b$ là số nguyên tố nếu trừ khả năng $a+b=2$ ra thì $a+b$ lẻ do đó $a,b$ có 1 số chẵn và 1 số lẻ khi đó làm sao bạn có được $a+b=ab$ với 2 vế khác nhau về tính chẵn lẻ?

 




#619298 Đề chọn đội tuyển trường Ams vòng 2-lần 2 năm 2015-2016

Đã gửi bởi quanganhthanhhoa on 09-03-2016 - 11:41 trong Tài liệu - Đề thi

1.2 Ta giải như sau 
Giả sử $ab-cd=a+b+c+d=k$ là một số nguyên tố 
Suy ra $a-1=k-b-c-d$ 
Sau một hồi thế vào cái trên cho ta $k=(d+b).\frac{c+b}{b-1} \in \mathbb{Z}$ 

Suy ra $c+1 \vdots b-1$ 
Tương tự c/m cũng được $c+1 \vdots a-1$ 
Từ đó suy ra $q(k+1)=1$ vô lí với $q \in \mathbb{N*}$ 
Suy ra đpcm

Bạn giải thích rõ giúp mình chỗ tô đỏ :)




#619301 Đề chọn đội tuyển trường Ams vòng 2-lần 2 năm 2015-2016

Đã gửi bởi quanganhthanhhoa on 09-03-2016 - 11:51 trong Tài liệu - Đề thi

bạn tự biến đổi đi

Không! Mình còn thắc mắc tại sao bạn đặt $a+b+c+d=k$ mà lại suy ra $a-1=k-b-c-d$ 

Với lại mình không hiểu mới hỏi,sao lại nói tự biến đổi được  :angry:




#607830 $2(x+y+z)-xyz\leq 10$

Đã gửi bởi quanganhthanhhoa on 07-01-2016 - 21:21 trong Bất đẳng thức và cực trị

1. Cho x,y,z là các số thực thỏa mãn  $x^{2}+y^{2}+z^{2}=9,xyz\leq 0$

 

CMR: $2(x+y+z)-xyz\leq 10$ 

 

Áp dụng BĐT Bunhiacopxki

$\left [2(x+y)+z(2-xy) \right ]^2\leqslant \left [ (x+y)^2+z^2 \right ](4+(2-xy)^2)$

$=(9+2xy)(8+x^2y^2-4xy)$

Ta sẽ đi cm $(9+2t)(8+t^2-4t)\leqslant 100$

$(t+2)^2(2t-7)\leqslant 0$ (với $t=xy$) $(*)$

Đến đây giả sử $|x|\leqslant |y|\leqslant |z|\Rightarrow x^2\leqslant y^2\leqslant z^2\Rightarrow x^2+y^2\leqslant 6$

Mà $xy\leqslant \frac{x^2+y^2}{2}\leqslant 3$ suy ra đpcm




#607998 TOPIC ĐỀ THI HSG LỚP 9

Đã gửi bởi quanganhthanhhoa on 08-01-2016 - 19:11 trong Tài liệu - Đề thi

 

Câu 3:

 

Giải phương trình sau:$4x^2-11x+10=(x-1)\sqrt{2x^2-6x+2}$

 

Câu 4:Cho tam giác ABC cân tại A có M là trung điểm của BC,O là điểm nằm trong tam giác sao cho $\widehat{OBA}=\widehat{OCB}$.Chứng minh rằng:$\widehat{AOC}+\widehat{BOM}=180^{\circ}$

4.Bài hình sao nghèo nàn quá vậy,đi thi hình luôn là câu gây trở ngại khó nhất cho thí sinh mà,bài này dễ quá :(

$\Delta OMB=\Delta OMC(c.g.c)\Rightarrow \widehat{BOM}=\widehat{CMO}\Rightarrow \widehat{BOM}+\widehat{AOC}=\widehat{CMO}+\widehat{AOC}=180^{\circ}\rightarrow Q.E.D$




#607999 TOPIC ĐỀ THI HSG LỚP 9

Đã gửi bởi quanganhthanhhoa on 08-01-2016 - 19:16 trong Tài liệu - Đề thi

Nhận xét về cấu trúc ra đề:

Phần đại số chiếm quá nhiều trong khi phần hình lại quá ít,và bài hình duy nhất ấy quá dễ,học sinh đại trà lớp 7 cũng làm được.

Bài bất đẳng thức không phù hợp với THCS khi lời giải phải dùng đến bất đẳng thức Schur.

Bài hệ khá ổn,bài phương trình vô tỉ nghiệm xảy ra THCS chưa giải được.

Tóm lại đề còn có nhiều sai sót và cần chỉnh sửa nhiều hơn nữa :)




#606959 Chứng minh rằng $m, n$ đều là lập phương của một số.

Đã gửi bởi quanganhthanhhoa on 03-01-2016 - 15:08 trong Số học

Giải

Giả sử $k=p_1^{a_1}.p_2^{a_2}...p_n^{a_n}$

$=> k^3 = p_1^{3a_1}.p_2^{3a_2}...p_n^{3a_n} $

Xét $p$ là 1 ước bất kì của $k$

Ta có $m.n \vdots p^{3}$

Mặt khác do $(m,n)=1 => m \vdots p^3  \text{hoặc } n \vdots p^3$ (vì nếu $n=p;m=p^2$ thì vô lý, tương tự trường hợp kia )

Từ đó, cứ như thế, $m,n$ đều là lập phương của số tự nhiên

Lập luận cho tương tự cho trường hợp tổng quát

Đoạn màu đỏ bạn làm thế nào để chứng minh là lập phương của số tự nhiên được,bạn phải chứng minh được $p^3 \vdots m$ và $p^3 \vdots n$ thì mới suy ra được lập phương của số tự nhiên chứ.Lập luận của bạn quá là mơ hồ đi !!




#608718 Cô gái đặc biệt

Đã gửi bởi quanganhthanhhoa on 12-01-2016 - 22:49 trong Quán văn

Chà,nghe như ngôn tình ấy nhỉ  :D  Cơ mà hay đó  :like




#607775 Tìm min $M=\frac{a^{5}}{b^{3}+c^{2}}+\frac{b^{5}}{c^{3}+a^{2}}...

Đã gửi bởi quanganhthanhhoa on 07-01-2016 - 17:52 trong Bất đẳng thức và cực trị

Áp dụng bất đẳng thức Cauchy-Schwarz ta có $ab^{2}+bc^{2}+ca^{2}\leq \sqrt{(a^{2}b^{2}+b^{2}c^{2}+c^{2}a^{2})(a^{2}+b^2+c^2)}\leq \sqrt{\frac{(a^2+b^2+c^2)^{2}}{3}.3}=3;a^{2}+b^2+c^2\leq \sqrt{3(a^{4}+b^4+c^4)}\Leftrightarrow 3\leq \sqrt{3(a^{4}+b^4+c^4)}\Leftrightarrow a^{4}+b^4+c^4\geq 3$




#603029 $\left\{\begin{matrix} x+y+z>11 &...

Đã gửi bởi quanganhthanhhoa on 13-12-2015 - 20:22 trong Số học

Đề là + 10z thì mới làm được chứ nhỉ ???

Mình sửa rồi đó,bạn giải đi




#602999 $\left\{\begin{matrix} x+y+z>11 &...

Đã gửi bởi quanganhthanhhoa on 13-12-2015 - 17:16 trong Số học

Tìm x,y,z nguyên dương thỏa mãn $\left\{\begin{matrix} x+y+z>11 & \\ 8x+9y+10z=100 & \end{matrix}\right.$




#607011 $P=\frac{1}{(1+x)^2}+\frac{1}...

Đã gửi bởi quanganhthanhhoa on 03-01-2016 - 18:35 trong Bất đẳng thức và cực trị

Cho $x,y,z$ là 3 số thực dương thỏa mãn: $xyz=1$. Tìm giá trị nhỏ nhất của biểu thức: $P=\frac{1}{(1+x)^2}+\frac{1}{(1+y)^2}+\frac{4}{3(z+1)^3}$

Có thể áp dụng bổ đề NÀY

Giả sử $z=max(x,y,z)\Rightarrow z\geq 1$ 

Áp dụng bổ đề trên kết hợp vs bất đẳng thức C-S ta có $P\geq \frac{1}{1+xy}+\frac{4}{3(z+1)^3}=\frac{z}{z+1}+\frac{4}{3(z+1)^3}=\frac{3z(z+1)^{2}+4}{3(z+1)^3}=\frac{3z^{3}+6z^2+3z+4}{3z^{3}+9z^2+9z+3}=1-\frac{z(z+1)}{(z+1)^{3}}-\frac{2}{3(z+1)^{3}}=1-\frac{z}{(z+1)^{2}}-\frac{2}{3(z+1)^{3}}\geq 1-\frac{z}{4z}-\frac{2}{3(z+1)^{3}}=\frac{3}{4}-\frac{2}{3(z+1)^{3}}\geq \frac{3}{4}-\frac{2}{3.2^{3}}=\frac{2}{3}$

Dấu ''='' xảy ra khi $x=y=z=1$




#612273 Cho $a,b,c>0$ và $a+b+c=3$.Chứng minh rằng: $...

Đã gửi bởi quanganhthanhhoa on 01-02-2016 - 18:18 trong Bất đẳng thức và cực trị

Cho $a,b,c>0$ và $a+b+c=3$.Chứng minh rằng:
$\frac{a^3}{(2a^2+b^2)(2a^2+c^2)}+\frac{b^3}{(2b^2+c^2)(2b^2+a^2)}+\frac{c^3}{(2c^2+a^2)(2c^2+b^2)}\leqslant \frac{1}{3}$

Đã có tại đây




#608222 $x^{2}-3\vdots xy+3$

Đã gửi bởi quanganhthanhhoa on 09-01-2016 - 21:41 trong Số học

Tìm x, y nguyên dương thỏa mãn:

(x2 -3) chia hết cho (xy+3)

$x^{2}-3\vdots xy+3\Rightarrow y(x^2-3)\vdots xy+3\Rightarrow x(xy+3)-3(x+y)\vdots xy+3\Leftrightarrow 3(x+y)\vdots xy+3\Leftrightarrow 3(x+y)\geq xy+3\Leftrightarrow (3-x)y\geq 3-3x$

Xét $x=1,2$ rồi tìm y

Xét $x \geq 3$ ta có $y\leq \frac{3x-3}{x-3}=3+\frac{6}{x-3}\leq 9\Rightarrow 1\leq y\leq 9$

Xét các trường hợp của $y$ để tìm $x$

Hơi nhiều trường hợp nhỉ =))




#608170 \[\frac{k}{a^{3}+b^{3}}+...

Đã gửi bởi quanganhthanhhoa on 09-01-2016 - 17:45 trong Bất đẳng thức và cực trị

Cho các số thực dương a,b. Tìm hằng số k lớn nhất thỏa mãn bất đẳng thức : \[\frac{k}{a^{3}+b^{3}}+\frac{1}{a^{3}}+\frac{1}{b^{3}}\geq \frac{16+4k}{\left ( a+b \right )^{3}}\]  

 

Biến đổi BĐT đã cho thành
$ ( \frac{k}{a^{3}+b^{3}}-\frac{4k}{(a+b)^{3}}  )+ ( \frac{1}{a^{3}}+\frac{1}{b^{3}}-\frac{16}{(a+b)^{3}})\geq 0$
$\Leftrightarrow ( \frac{a-b}{a+b})^{2}[ \frac{(a^{2}+ab+b^{2})^{2}+3ab(a+b)^{2}+3a^{2}b^{2}}{a^{3}b^{3}(a+b)}-\frac{3k}{a^{3}+b^{3}} ]\geq 0$
Nên BĐT sau phải đúng:
$ [ (a^{2}+ab+b^{2})^{2}+3ab(a+b)^{2}+3a^{2}b^{2} ](a^{3}+b^{3})\geq 3ka^{3}b^{3}(a+b)$
Cho $a=b$ suy ra $k\leq 8$. Mặt khác, khi k=8 thì theo AM-GM:
$(a^{2}+ab+b^{2})^{2}+3ab(a+b)^{2}+3a^{2}b^{2}\geq 24a^{2}b^{2}$
$a^{3}+b^{3}\geq ab(a+b)$ (đpcm)
Hằng số $k$ tốt nhất là $k=8$




#608171 $\left\{\begin{matrix} \sqrt{...

Đã gửi bởi quanganhthanhhoa on 09-01-2016 - 17:54 trong Đại số

Giải hệ phương trình

 $\left\{\begin{matrix} \sqrt{\frac{6x}{x+y}} + \sqrt{\frac{x+y}{6x}} = \frac{5}{2} \\ x+y-xy = 9 \end{matrix}\right.$

ĐK:$x>0;x>-y$

Đặt $\sqrt{\frac{6x}{x+y}} =a\Rightarrow  \sqrt{\frac{x+y}{6x}} =\frac{1}{a}\Rightarrow a+\frac{1}{a}=\frac{5}{2}\Leftrightarrow 2(a^{2}+1)=5a\Leftrightarrow (2a-1)(a-2)=0\Leftrightarrow \begin{bmatrix}a=\frac{1}{2}\Leftrightarrow \frac{6x}{x+y}=\frac{1}{4}\Leftrightarrow 23x=y & \\ a=2\Leftrightarrow\frac{6x}{x+y}=4\Leftrightarrow x=2y   & \end{bmatrix}$

Thay vào phương trình (2) bạn sẽ tìm được nghiệm $x,y$